조화 진동자: 두 판 사이의 차이

내용 삭제됨 내용 추가됨
몇가지 내용 추가가가가가가가가
1번째 줄:
*
{{알찬 글}}
{{다른 뜻|양자 조화 진동자|고전역학|양자역학}}
[[파일:2006-02-04 Metal spiral.jpg|right|300px|thumb|조화 진동을 하는 대표적인 예인 [[용수철]].]]
'''조화 진동자'''(調和振動子, {{llang|en|harmonic oscillator}})는 [[고전역학]]에서 다루는 기본적인 [[계 (물리학)|계]] 중의 하나로, 평형점에서 물체가 이동했을 때, [[훅 법칙]]에 의한 [[복원력]]
:<math>F = - kx</math>
을 받는 계이다. 여기서 <math>k</math>는 양의 [[상수]]이다.
 
만약, <math>F</math>가 계에 작용하는 유일한 힘이라면 이 진동자를 '''단순 조화 진동자'''({{lang|en|simple harmonic oscillator}})라 한다. 이 계의 운동은, [[진폭]]과 [[진동수]]가 일정한 [[사인파|사인 모양]] [[진동]]을 보여준다.
 
[[속도]]에 비례하는 [[마찰력]]이 존재하는 경우에는 이 진동자를 '''감쇠 진동자'''({{lang|en|damped oscillator}})라 한다. 이 경우에는, 마찰이 없는 경우에 비해 진동수가 작아지고 진폭 또한 시간에 따라 점점 줄어드는 운동을 보인다.
 
마지막으로, 마찰력이 아닌 다른 [[외력]]이 이 계에 작용하는 경우에는 이 진동자를 '''강제 진동자'''({{lang|en|forced oscillator}})라 한다.
 
이러한 진동자의 역학적 예로는 [[질량]]이 있는 물체가 연결된 [[용수철]], 작게 진동하는 [[진자]] 그리고 기타의 현과 같은 음향계들이 있다. 또한, 이와 유사한 행동을 보여주는 [[RLC 회로]]와 같은 전기적인 조화진동자도 있다. 실제로 자연이나 인공적으로 만들어진 진동에는 이상적이고 완전한 조화진동자는 없지만, 조화진동자를 분석하면 수학과 물리학, 그리고 여러 응용과학에서 자연의 여러 계에 대해 깊은 이해를 하는데 도움을 준다.
 
이 문서에서는 [[고전역학]]의 표기법을 따라 시간에 대한 미분은 변수 위에 점을 찍어 표현한다.
 
====== 단순 조화 진동 ======
[[파일:Simple harmonic oscillator.gif|right|100px|thumb|감쇠가 없는 [[용수철]]의 진동.]]
[[훅 법칙]]에 의한 힘 이외에 다른 힘을 받지 않는 진동을 '''단순 조화 진동''' (간단히 '''단진동''') 또는 '''자유 진동'''({{lang|en|free oscillation}})이라고 한다. 보통 일직선상에서 주기적이며, 사인 모양의 운동을 보인다. 이 운동은 경우에 따라 서로 다른 [[진동수]], [[주기 (물리학)|주기]], [[진폭]], [[위상]]을 가지게 된다. 여기서 진동수와 주기는 계의 구성에 따라 바뀌게 되며, 진폭과 위상은 [[초기 조건]]에 따라 바뀌게 된다.
 
=== 운동 방정식 ===
단순 조화 진동의 [[운동 방정식]]은 다음과 같이 주어진다.
:<math>m \ddot{x} + kx = 0</math>
보통 여기서 &omega;<sub>0</sub>를 다음과 같이 정의하여
:<math>\omega_0^2 = { k \over m}</math>
운동 방정식을 다음과 같이 쓴다.
:<math>\ddot{x} + \omega_0^2 x = 0</math>
이 방정식의 해는 다음과 같다.<ref>{{서적 인용 |저자= Dennis. G. Zill|공저자= Michael R. Cullen|제목= Advanced Engineering Mathematics|연도=2006|출판사= Jones & Bartlett Pub|판=Third Edition|쪽=p. 120}}</ref>
:<math>x(t) = C_1 \sin \omega_0 t + C_2 \cos \omega_0 t</math>
여기서 <math>C_1</math>와 <math>C_2</math>는 상수로 초기 조건에 따라 결정되는 값이다. 좀 더 식에 물리학적 의미를 부여하기 위해 다음과 해를
:<math>x(t) = A \cos \left( \omega_0 t + \phi \right)</math>
::( 또는 <math>x(t) = A \sin \left( \omega_0 t + \phi \right)</math> )
로 나타내기도 한다. 여기서 <math>A</math>는 [[진폭]], <math>\phi</math>는 진동의 [[위상]]을 의미하는 상수로 위와 마찬가지로 초기 조건에 따라 결정되는 값이다. 이 식에서 <math>\omega_0</math>의 물리적 의미는 주기 운동의 [[각진동수]]에 해당한다. 그리고 이 운동의 속도와 가속도는 다음과 같다.
:<math>v(t) = -A\omega_0 \sin \left( \omega_0 t + \phi \right)</math>
:<math>a(t) = -A\omega_0^2 \cos \left( \omega_0 t + \phi \right)</math>
이 운동의 주기 <math>T</math>는 다음과 같이 주어진다.<ref name="step102">{{서적 인용 |저자= Stephen T. Thornton|공저자= Jerry B. Marion|제목= Classical Dynamics of Particles and Systems|연도=2003|출판사= Brooks/Cole|판=Fifth Edition|쪽=p. 102|장=3.2 Simple Harmonic Oscillator}}</ref>
:<math>T = {1 \over f} = {2 \pi \over \omega_0} = 2 \pi \sqrt{m \over k}</math>
이 운동의 [[운동 에너지]] <math>K</math>는 다음과 같이 주어진다.<ref name="step101">{{서적 인용 |저자= Stephen T. Thornton|공저자= Jerry B. Marion|제목= Classical Dynamics of Particles and Systems|연도=2003|출판사= Brooks/Cole|판=Fifth Edition|쪽=p. 101|장=3.2 Simple Harmonic Oscillator}}</ref>
 
:<math>\begin{align}
K = {1 \over 2} m \dot{x}^2 & = {1 \over 2} m A^2 \omega_0^2 \sin^2 \left( \omega_0 t + \phi \right)
\\ & = {1 \over 2} k A^2\sin^2 \left( \omega_0 t + \phi \right)
\end{align} </math>
[[위치에너지]] <math>U</math>는 <math>F = - \nabla U</math>에 의해 다음과 같이 주어진다.<ref name="step101" />
:<math>U = {1 \over 2} k x^2 = {1 \over 2} k A^2 \cos^2 \left( \omega_0 t + \phi \right)</math>
따라서 단순 조화 진동의 [[역학적 에너지]] E는 다음과 같이 주어진다.<ref name="step101" />
:<math>E = K + U = {1 \over 2} k A^2 </math>
 
=== 상도표 ===
[[파일:Phase_diagram_simple_ho.png|right|thumb|300px|단순조화진동에서 [[에너지]]가 다른 경우 몇 가지를 나타낸 [[상도표]]]]
단순 조화 진동의 운동은 위치 <math>x(t)</math>와 속도 <math>\dot{x}(t)</math>로 기술되는 [[위상 공간 (물리학)|위상 공간]]을 통해서도 나타낼 수 있다. 단순 조화 진동은 다음과 같이 위치와 속도로 기술할 수 있다.
:<math>x(t) = A \cos \left( \omega_0 t + \phi \right)</math>
:<math>\dot{x}(t) = -A\omega_0 \sin \left( \omega_0 t + \phi \right)</math>
여기서 시간 <math>t</math>를 소거하면,
:<math>{x^2 \over A^2} + {\dot{x}^2 \over A^2 \omega_0^2 } = 1</math>
이 되어 이 운동은 위상 공간에서 타원 모양으로 기술되는 운동임을 알 수 있다. 여기에 이 운동의 전체 에너지 <math>\textstyle E = {1\over 2} kA^2</math>를 대입하면
:<math>{x^2 \over {2E \over k}} + {\dot{x}^2 \over {2E \over m} } = 1</math>
이 되어 위상 공간에서 나타나게 되는 타원은 각각 특정 에너지를 갖는 운동을 기술함을 알 수 있다.<ref>{{서적 인용 |저자= Stephen T. Thornton|공저자= Jerry B. Marion|제목= Classical Dynamics of Particles and Systems|연도=2003|출판사= Brooks/Cole|판=Fifth Edition|쪽=p. 106-8|장=3.4 Phase Diagrams}}</ref>
 
=== 예 ===
==== 용수철 ====
{{참고|용수철}}
[[파일:Harmonic oscillator.svg|thumb|right|250px|[[용수철]]과 [[질량]]이 있는 물체로 이루어진 [[계 (물리학)|계]]. (A)는 평형점, (B)는 압축된 상태, (C)는 당겨진 상태의 힘의 방향을 보여준다.]]
 
용수철의 운동은 단순 조화 운동의 가장 대표적인 예이다. [[용수철]]이 망가지지 않는 범위에서 질량이 <math>m</math>인 물체를 사용해 용수철을 변형시키면 [[훅 법칙]]에 의한
:<math>F = - kx</math>
꼴의 [[복원력]]을 받는다. 여기서 <math>F</math>는 용수철에 의한 힘, <math>k</math>는 용수철 상수, <math>x</math>는 평형점으로부터 용수철이 변형된 변위이다. 따라서 이 힘을 제외한 다른 외력이 작용하지 않는 경우, 용수철은 단순 조화 진동을 하게 된다.
 
구체적으로는 [[뉴턴의 제2법칙]]을 사용해
:<math>F = ma = -kx</math>
이 물체의 운동에 대한 운동방정식을 만들 수 있고, 초기에 <math>A</math>에 자리 잡고 있고 정지해 있으면 이 운동의 해는 다음과 같이
:<math> x \left( t \right) =A\cos \textstyle \sqrt {k \over m} t</math>
[[코사인]]함수로 나타나며, 따라서 이 물체는 용수철에 의해 좌우로 진동함을 알 수 있다.
 
==== 진자 ====
{{본문|진자}}
[[진자]]의 운동은 단순 조화 운동이 아니지만, [[작은각 근사]]를 통하면 단순조화운동으로 근사될 수 있다. <math>I</math>를 진자의 [[관성모멘트]] (여기서는 <math>I = m \ell^2</math>. ), m을 진자 끝에 달린 물체의 질량, <math>\ell</math>를 끈의 길이라 하자. 이 운동을 돌림힘 &tau;를 통해 분석해보면,
:<math>\tau = \ell (mg \sin \theta) = I \ddot{\theta}</math>
이 되고, (<math>I</math>는 [[관성모멘트]], 여기서는 <math>I = m \ell^2</math>. ) 작은각 근사
:<math>\sin \theta \approx \theta </math>
를 쓰면
:<math>\ell mg \theta \approx I \ddot{\theta}</math>
가 된다. 단순 조화 운동의 방정식과 위 식을 비교해보면 정확히 일치함을 알 수 있다.
 
여기서 이 진자의 주기는 줄의 길이 <math>\ell</math>와 [[중력 가속도]] <math>g</math>와 관련이 있다. 그리고 위의 용수철에 대한 주기와 비슷한 형태를 보이고 있다.
 
:<math> T= 2 \pi \sqrt{\frac{\ell}{g}}</math>
 
[[파일:ComplexSinInATimeAxe.gif|right|thumb|300px|[[원운동]]을 직교좌표계의 x축으로 [[사영]]시키면 [[사인 모양]]의 단진동이 나타난다.]]
 
==== 등속 원운동 ====
{{본문|등속원운동}}
단순 조화 진동은 등속 원운동의 1차원 사영으로 볼 수도 있다. 어떤 물체가 각진동수 <math>\omega</math>로 반지름이 <math>R</math>인 xy평면 위의 원에서 원운동을 하면 이 운동의 x좌표와 y좌표는 진폭이 <math>R</math>이고 각진동수가 <math>\omega</math>인 단순 조화 운동의 경우와 똑같은 방정식이 된다. 각속도가 <math>\omega_0</math>, 초기 위치가 극좌표에서 <math>(A,\phi)</math>인 등속 원운동을 2차원 [[극좌표]]에 표현하면
:<math>r(t) = A</math>
:<math>\theta(t) = \omega_0 t + \phi</math>
인 운동이 되는데 이를 [[직교좌표계]]로 변환시켜 보면
:<math>x = A \,\cos ( \omega_0 t + \phi)</math>
:<math>y = A \,\sin ( \omega_0 t + \phi)</math>
가 되어 이를 쉽게 확인할 수 있다.
 
=== 해밀토니언과 정준 변환을 사용한 운동의 풀이 ===
단순 조화 진동의 [[해밀토니언]]은 다음과 같이 주어진다.
:<math>H = {p^2 \over 2m } + {1 \over 2} k q^2</math>
여기서 <math>p</math>는 물체의 운동량, <math>q</math>는 평형점으로부터 이동한 거리를 의미한다. 이를 <math>\omega_0</math>를 통해 쓴 후, 식을 다시 쓰면
:<math>H = \frac{1}{2m} \left( p^2 + m^2 \omega_0^2 q^2 \right)</math>
이다. 위 식을 자세히 살펴보면 두 항이 서로 제곱 형태로 되어 있음을 알 수 있다. 따라서 새로운 좌표 <math>(P,Q)</math>로 다음과 같은 꼴의 [[정준 변환]]을 찾을 수 있다면,
:<math>p = f(P)\,\cos Q</math>
:<math>q = \frac{f(P)}{m \omega_0} \sin Q </math>
이 계의 해밀토니언은 다음과 같이 간단한 꼴이 되고
:<math>H = \frac{f(P)^2}{2m} (\cos^2 Q + \sin^2 Q ) = \frac{f(P)^2}{2m}</math>
좌표 <math>Q</math>는 [[순환좌표]]가 된다. 다만, 이 방법의 문제점은 어떻게 <math>f(P)</math>를 결정하여 이 변환을 [[정준변환|정준]]이 되게 만들 수 있느냐이다. 다음과 같은 [[모함수 (물리학)|모함수]]를 사용하면,
:<math>F_1 = {m \omega_0 q^2 \over 2} \cot Q </math>
아래의 변환에 대한 방정식을 얻을 수 있다,
:<math> p = \frac{\partial F_1 }{\partial q} = m \omega_0 q \cot Q</math>
:<math> P = -\frac{\partial F_1 }{\partial Q} = \frac{m \omega_0 q^2}{2 \sin^2 Q}</math>
이를 <math>q</math>와 <math>p</math>에 대해 풀면,
:<math>p = \sqrt{2Pm\omega_0} \cos Q</math>
:<math>q = \sqrt{\frac{2P}{m \omega_0}} \sin Q </math>
을 얻는다. 따라서 함수 <math>f(P)</math>는
:<math>f(P) = \sqrt{2m \omega_0 P}</math>
이다. 이를 해밀토니언에 대입하면 이 새로운 좌표에 대한 해밀토니안은
:<math>H = \omega_0 P</math>
가 된다. 여기서 좌표 <math>Q</math>가 [[순환 좌표]]이기 때문에 그에 해당하는 [[일반화 운동량|운동량]]인 <math>P</math>는 [[운동상수]]가 된다. 따라서
:<math>P = {E \over \omega}</math>
가 되고, <math>Q</math>의 운동 방정식, 즉 [[해밀턴 방정식]]은
:<math>\dot{Q} = \frac{\partial H } {\partial P} = \omega_0</math>
가 되어 간단히 이 운동의 해는
:<math>Q = \omega_0 t + \alpha</math>
가 된다. 여기서 <math>\alpha</math>는 임의의 상수이다. 이제 역변환을 통해 좌표 <math>(p,q)</math>에서의 해를 구해보면
:<math>q = \sqrt{\frac{2E}{m \omega_0^2 }} \sin (\omega_1 t + \alpha)</math>
:<math>p = \sqrt{2mE} \cos (\omega_1 t + \alpha )</math>
이다. 그리고 이에 대한 [[위상 공간 (물리학)|위상 공간]] <math>(p,q)</math> 에서의 상도표는 자연스럽게 타원이 됨을 알 수 있다.<ref>{{서적 인용 |저자=Herbert Goldstein|공저자=Charles Poole, John Safko|제목= Classical Mechanics|연도=2002|출판사=Addison Wesley|판=Third Edition|쪽=p. 377-9|장=9.3 The Harmonic Oscillator}}</ref><ref>문희태(2006), 《개정판 고전역학》, 서울 : 서울대학교출판부, 291-2쪽</ref>
 
== 강제 진동 ==
단순 조화 진동에서 [[계 (물리학)|계]]에 시간에 대한 임의의 함수로 표현되는 외력이 작용한 경우, 이러한 진동을 '''강제 조화 진동''' 또는 간단히 '''강제 진동'''({{lang|en|forced oscillation, driven oscillation}})이라고 한다. [[고막]]·[[피아노]]의 울림판 등은 강제 진동의 예이다.<ref>'강제진동 - 음의 지식', 《글로벌 세계 대백과》</ref>
 
=== 운동 방정식 ===
강제진동의 [[운동 방정식]]은 외력을 <math>F(t)</math>라 하면 다음과 같은 비동차 [[미분 방정식]]으로 주어진다.
:<math>m \ddot{x} + kx = F(t)</math>
이 운동의 해는 위 방정식의 동차 미분 방정식 부분인 단순 조화 진동의 일반해 <math>x_h(t)</math>와 외력 <math>F(t)</math>와 관계있는 특수해 <math>x_p(t)</math>의 합으로 준다.<ref>{{서적 인용|저자=문희태|연도=2006|제목=개정판 고전역학|위치=서울|출판사=서울대학교출판부|쪽=174}}</ref>
:<math>x(t) = x_h (t) + x_p (t)</math>
 
==== 외력이 주기적 힘일 때 ====
여기서 외력이 다음과 같이
:<math>F(t) = B cos (\omega_1 t + \alpha) </math>
주기적으로 주어지는 경우 [[공명]]이란 특이한 현상이 나타난다. 먼저 특수해를 구해보면
:<math>x_p (t) = {B \over m (\omega_0^2 - \omega_1^2) } \cos (\omega_1 t + \alpha)</math>
가 된다. 따라서, 이 경우 운동의 해는 다음과 같다.
:<math>x(t) = A \cos \left( \omega_0 t + \phi \right) + {B \over m (\omega_0^2 - \omega_1^2) } \cos (\omega_1 t + \alpha)</math>
여기서 외력의 각진동수 <math>\omega_1</math>가 단순조화운동의 각진동수 <math>\omega_0</math>에 가까워 지면 진동의 진폭이 점점 커짐을 알 수 있다.<ref>{{서적 인용|저자=문희태|연도=2006|제목=개정판 고전역학|위치=서울|출판사=서울대학교출판부|쪽=175}}</ref>
<!-- 여기서 <math>\omega_1</math>가 <math>\omega_0</math>에 매우 가까워 지면 어떤 현상이 일어나는지 알아보기 위하여 -->
==== 외력이 일반적인 힘일 때 ====
외력이 시간에 따라 변하는 임의의 힘이라면, <math>\xi = \dot{x} + i \omega_0 x </math> 라는 양을 정의해 문제를 해결한다. 이때, 운동방정식은 다음과 같은 형태를 보인다.
:<math>\dot \xi - i \omega_0 \xi = {F(t) \over m}</math>
그리고 이 운동의 해는 위 미분 방정식을 풀고서, 허수부를 <math>\omega_0</math>로 나누어 주면 운동 방정식의 특수해 <math>x_p(t)</math>를 얻을 수 있다. 특수해를 <math>\xi = C(t) e^{i \omega_0 t}</math>라 하면, <math>C(t)</math>가 만족하게 해야 하는
 
다음과 같다.
:<math>\dot C = {F(t) \over m} e^{i\omega_0 t}</math>
위 방정식을 풀면, 이 운동의 특수해는 다음과 같아야 함을 알 수 있다.
:<math>\xi(t) = \left[ \int_0^t {{F(t') \over m } e^{-i \omega_0 t'} \, dt' } \right] e^{i \omega_0 t } + \xi_0</math>
여기서 <math>\xi_0</math>는 초기조건에 따라 결정되는 값이다.
 
이 경우, 시간에 따라 변하는 외력이 존재하기 때문에 에너지는 보존되지 않는다. 이를 통해 계에 전달되는 에너지의 양은 초기에 진동이 멈춰 있는 경우 (즉, <math>x = 0</math> , <math>\dot{x} = 0</math>. 그러므로, <math>x_h (t) = 0</math> 이고 <math>\xi_0 = 0</math> 이다.) 계의 총 에너지 E 는 다음과 같이 <math>\xi</math>를 사용하여 나타낼 수 있다.
:<math>E = {1 \over 2} m (\dot{x}^2 + \omega_0^2 x^2 ) = {1 \over 2 } m | \xi |^2</math>
힘이 작용하지 않았을 때를 <math>F(t) = 0 </math>라 정의하면, 전달된 에너지는 다음과 같이 쓸 수 있다.
:<math>E = {1 \over 2m } \left| \int_{-\infty}^{\infty} {F(t) e^{-i \omega t} \, dt}\right|^2</math>
맨 뒤 항의 적분은 외력의 [[푸리에 변환]]에 해당하므로, 여기서 유입된 에너지는 외력에 포함된 여러 진동수 중, 단순조화진동일 때의 진동수 성분 크기의 제곱에 의해 결정된다.<ref>문희태(2006), 《개정판 고전역학》, 서울 : 서울대학교출판부, 177-8쪽</ref>
 
== 감쇠 진동 ==
[[파일:Damped spring.gif|right|100px|thumb|감쇠가 있는 [[용수철]]의 진동. 이 경우에는 저감쇠 진동을 하고 있다.]]
[[속도]]에 비례하는 [[마찰력]]이 존재할 경우, 이러한 조건에서의 진동을 '''감쇠 진동'''({{lang|en|damped oscillation}})이라 한다. 실제 이상적이 아닌 상황에선 항상 마찰이 존재하기 때문에 모든 진동은 감쇠 진동을 한다고 볼 수 있다.
 
=== 운동 방정식 ===
감쇠 진동의 경우, 다음과 같은 속도에 비례하는 마찰력
:<math>F = -b \dot{x}</math>
가 있기 때문에, 운동 방정식은 이를 포함하는 방정식이 된다.
:<math>m \ddot{x} + b \dot{x} + kx = 0</math>
이 식을 질량 <math>m</math>으로 나누고, <math>\textstyle 2\lambda = {b \over m} </math> , <math>\textstyle \omega_0^2 = {k \over m}</math>라 놓으면 위 식은 다음과 같은 식이 된다.
:<math>\ddot{x} + 2 \lambda \dot{x} + \omega_0^2 x = 0</math>
위의 미분 방정식은 <math>\textstyle e^{ct} </math> 꼴의 해가 항상 존재하는 것으로 알려졌다. 이를 위에 대입하면 가능한 상수 <math>c</math>의 값은
:<math>c = - \lambda \pm \sqrt{\lambda^2 - \omega_0^2}</math>
이 된다. 여기서 <math>\lambda</math>와 <math>\omega_0</math>의 값에 따라 위 근이 두 개의 실근, 중근, 두 개의 복소수근이 되는가가 결정된다. 여기서 두 개의 복소수근을 갖는 경우는 '''저감쇠 진동'''({{lang|en|underdamped oscillation}}), 두 개의 실근을 갖는 경우를 '''과감쇠 진동'''({{lang|en|overdamped oscillation}}), 마지막으로 중근을 갖는 경우를 '''임계 감쇠 진동'''({{lang|en|critically damped oscillation}})이라 한다.
==== 저감쇠 진동 ====
[[파일:Underdamped oscillation xt.png|right|thumb|350px|<math>\lambda = 2/\textrm{s}</math> , <math>\omega_1 = 20/\textrm{s}</math>인 경우의 저감쇠 진동의 시간에 대한 진폭의 그래프(빨강). 파란색 선을 따라 점점 지수적으로 감소하는 진동을 볼 수 있다. 초기 조건은 <math>x(0) = 1\textrm{m}</math> , <math>\dot{x}(0) = 0</math>.]]
[[파일:Over n crit damped osc xt.png|right|thumb|350px|<math>\lambda = 13/\textrm{s}</math> , <math>\omega_2 = 10/\textrm{s}</math>인 경우의 과감쇠진동(파랑)과 <math>\lambda = 13/\textrm{s}</math>인 경우의 임계 감쇠 진동 (초록)의 시간에 대한 진폭의 그래프. 지수적으로 급격히 감소하는 진동을 볼 수 있다. 초기조건은 <math>x(0) = 1\textrm{m}</math> , <math>\dot{x}(0) = 0</math>.]]
 
<math> \lambda < \omega_0</math>인 경우를 '''저감쇠 진동''' 또는 '''주기적 감쇠 진동'''({{lang|en|underdamped oscillation}})이라 한다. 보통 공기 속에서의 진동과 같이 마찰이 비교적 적었을 때 이러한 진동이 나타난다. 이 경우, 근이 다음과 같이 복소수이기 때문에
:<math>x(t) = c_1 e^{- \lambda t + i \sqrt{\omega_0^2 - \lambda^2}t } + c_2 e^{- \lambda t - i \sqrt{\omega_0^2 - \lambda^2}t }</math>
물리학적 의미를 부여하기 위하여 식을 조절하여 다음과 같이 근을 쓴다.<ref name="zill154">{{서적 인용 |저자= Dennis. G. Zill|공저자= Michael R. Cullen|제목= Advanced Engineering Mathematics|연도=2006|출판사= Jones & Bartlett Pub|판=Third Edition|쪽=p. 154}}</ref>
:<math>x(t) = a e^{- \lambda t} \cos ({\omega_1 t + \alpha} ) \,</math>
여기서, <math>a</math>와 <math>\alpha</math>는 임의의 상수이고,
:<math> \omega_1 = \sqrt{\omega_0^2 - \lambda^2} </math>
이다. 이 경우, 같은 조건의 단순 조화 운동에 비해 각진동수가 낮고, 진폭이 시간이 지남에 따라 점점 줄어드는 것을 알 수 있다.<ref name="zill154" />
 
==== 과감쇠 진동 ====
<math> \lambda > \omega_0</math>인 경우를 '''과감쇠 진동''' 또는 '''지수적 감쇠 진동'''({{lang|en|overdamped oscillation}})이라 한다. 물속과 같은 강한 저항이 존재하는 곳에서 진동이 일어날 때 많이 일어나는 진동이다. 이 경우, 근은 다음과 같이 실근으로 나온다.<ref name="zill154" />
:<math>x(t) = c_1 e^{- \lambda t + \sqrt{ \lambda^2 - \omega_0^2 }t } + c_2 e^{- \lambda t - \sqrt{\lambda^2 - \omega_0^2}t }</math>
좀 더 보기 쉽게 위 근을 아래와 같이 쓰기도 한다.
:<math>x(t) = e^{- \lambda t} \left[ A_1 e^{\omega_2 t} + A_2 e^{-\omega_2 t} \right]</math>
여기서 <math>A_1</math>과 <math>A_2</math>는 임의의 상수이고,
:<math> \omega_2 = \sqrt{\lambda^2 - \omega_0^2 } </math>
이다. 여기서 <math>\omega_2</math>는 <math>\omega</math>로 쓰긴 했지만, 이 운동은 주기 운동이 아니므로, 각진동수를 의미하는 값이 아님에 유의하자.
 
==== 임계 감쇠 진동 ====
<math> \lambda = \omega_0</math>인 경우를 '''임계 감쇠 진동'''({{lang|en|critically damped oscillation}})이라 한다. 이 진동은 저감쇠진동과 과감쇠진동의 중간에 있는 진동이다. 이 경우, 위에서 구한 근은 하나뿐이기 때문에 이 운동을 풀기 위해선 하나의 해가 더 필요하다. 그 해는 <math>t e^{-\lambda t}</math> 꼴의 해임이 알려졌다.<ref name="zill154" /> 따라서 이 운동의 해는 다음과 같다.<ref name="zill154" />
:<math>x(t) = (c_1 + c_2 t ) e^{-\lambda t} \;</math>
여기서 <math>c_1</math>과 <math>c_2</math>는 임의의 상수이다.
 
주어진 조건에서, 임계 감쇠 진동은 어떤 진동이 가장 빨리 멈추는 진동의 형태이다.<ref>{{서적 인용 |저자= Stephen T. Thornton|공저자= Jerry B. Marion|제목= Classical Dynamics of Particles and Systems|연도=2003|출판사= Brooks/Cole|판=Fifth Edition|쪽=p. 114|장=3.5 Damped Oscillations, Critically Damped Motion}}</ref>
 
=== 상도표 ===
==== 저감쇠 진동 ====
[[파일:Phase diagram under ho.png|right|thumb|350px|저감쇠진동의 상도표, 초기조건은 <math>A=1 \textrm{m}</math> , <math>\omega_0 = 1 \textrm{rad/s}</math> , <math>\lambda = 0.2 \textrm{s}^{-1}</math>, <math>\textstyle \alpha = - {\pi \over 2}</math>.]]
저감쇠진동의 경우, 시간에 따른 물체의 위치와 속도는 다음과 같이 나타난다.
:<math>x(t) = a e^{- \lambda t} \cos ({\omega_1 t + \alpha} ) \,</math>
:<math>\dot{x}(t) = - ae^{-\lambda t} \left[ \lambda \cos ({\omega_1 t + \alpha}) + \omega_1 \sin ({\omega_1 t + \alpha}) \right] \, </math>
이 두 변수에 의한 [[위상공간 (물리학)|위상 공간]]에서의 좌표는 다음과 같이 새로운 좌표 <math>(u, v)</math>로 [[선형변환]]을 하면 그 의미를 쉽게 이해할 수 있다.
:<math>u = \omega_1 x, \quad v = \lambda x + \dot{ x }</math>
위 두 좌표로 식을 다시 쓰면,
:<math>u = \omega a e^{- \lambda t} \cos ({\omega_1 t + \alpha} ) \,</math>
:<math>v = - \omega a e^{- \lambda t} \sin ({\omega_1 t + \alpha} ) \,</math>
두 식은 사인함수와 코사인 함수를 제외하고는 비슷한 형태로 나타내어져 있기 때문에 [[극좌표]]를 사용하면 이를 쉽게 기술할 수 있다. 다음과 같이 극좌표 <math>(\rho, \theta)</math>로 변환을 하면
:<math>\rho = \sqrt{u^2 + v^2}, \quad \theta = \omega_1 t</math>
아래와 같은 위상 공간에서 저감쇠 진동의 상도표를 나타내는 방정식을 구할 수 있다.
:<math>\rho = \omega_1 a e^{-{\lambda \over \omega_1} \theta } </math>
위를 보면, 이 운동은 위상 공간에서 안정한 나선을 그림을 알 수 있다.<ref>{{서적 인용 |저자= Stephen T. Thornton|공저자= Jerry B. Marion|제목= Classical Dynamics of Particles and Systems|연도=2003|출판사= Brooks/Cole|판=Fifth Edition|쪽=p. 111-3|장=3.5 Damped Oscillations, Underdamped Motion}}</ref>
 
==== 과감쇠 진동 ====
과감쇠 진동은 진동이 지수적으로 평형점으로 수렴하기 때문에, 이의 상도표는 평형점으로 급격히 떨어지는 모습을 취한다.
 
== 강제 감쇠 진동 ==
'''강제 감쇠 진동'''({{lang|en|forced damped oscillation, driven damped oscillation}})은 마찰력 외의 시간에 관계된 외력이 존재하는 경우의 감쇠 진동을 말한다.
=== 운동 방정식 ===
이 경우의 운동 방정식은 강제 진동과 마찬가지로, 우변에 외력에 관한 항 <math>F(t)</math>가 나타난다.
:<math>m \ddot{x} + b \dot{x} + kx = F(t)</math>
또한, 감쇠 진동과 마찬가지로 더 간단히 이 식을 아래와 같이 쓰기도 한다.
:<math> \ddot{x} + 2\lambda \dot{x} + \omega_0^2 x = {F(t) \over m }</math>
이 운동의 해는 강제 진동과 비슷하게 [[동차해]]에 해당하는 감쇠 진동의 해 <math>x_h(t)</math>와 외력에 따라 변하는 특수해 <math>x_p(t)</math>로 구성되어 있다.
:<math>x(t) = x_h(t) + x_p(t)</math>
 
==== 외력이 주기적 힘일 때 ====
외력이 다음과 같이 주기적으로 주어지는 경우를 생각해보자.
:<math>F(t) = F_0 \cos \omega_1 t</math>
이 경우 특수해는 다음과 같은 형태를 보인다.
:<math>x_p(t) = B \cos (\omega_1 t + \delta) </math>
이를 운동 방정식에 대입하고, 삼각함수를 전개하면,
:<math>\begin{align}
\left[ { F_0 \over m} - B \left\{ (\omega_0^2 - \omega_1^2 ) \cos \delta - 2 \lambda \omega_1 \sin \delta \right\} \right] \cos \omega_1 t
\\ + B \left[ (\omega_0^2 - \omega_1^2 ) \sin \delta + 2 \lambda \omega_1 \cos \delta \right] \sin \omega_1 t = 0
\end{align}</math>
이 된다. 두 삼각함수 <math>\sin \omega_1 t</math>와 <math>\cos \omega_1 t </math>는 [[선형독립]]인 함수이므로, 각 항의 계수의 값은 0이 되어야 한다. 먼저, <math>\sin \omega_1 t</math> 항에서 <math>\delta</math>에 대한 조건인
:<math> \tan \delta = \frac{2 \lambda \omega_1}{\omega_1^2 - \omega_0^2}</math>
을 얻을 수 있다. 이를 사인 함수와 코사인 함수에 대한 식으로 고치면
:<math> \sin \delta = { 2\lambda \omega_1 \over \sqrt{( \omega_1^2 - \omega_0^2 )^2 + 4 \lambda^2 \omega_1^2 }}</math>
:<math> \cos \delta = { \omega_1^2 - \omega_0^2 \over \sqrt{( \omega_1^2 - \omega_0^2 )^2 + 4 \lambda^2 \omega_1^2 }}</math>
이 된다. 이를 <math>\cos \omega_1 t </math>항에 대입하 상수 <math>B</math>를 구하면,
:<math> B = {F_0 \over m \sqrt{( \omega_1^2 - \omega_0^2 )^2 + 4 \lambda^2 \omega_1^2 } }</math>
을 얻는다. 따라서 이 경우의 특이해는
:<math> x_p(t) = {F_0 \over m \sqrt{( \omega_1^2 - \omega_0^2 )^2 + 4 \lambda^2 \omega_1^2 } } \cos (\omega_1 t + \delta) </math>
:<math> \delta = \tan^{-1} \frac{2 \lambda \omega_1}{\omega_1^2 - \omega_0^2}</math>
이다.
 
이 경우의 특이해는 여러 응용 분야에서 등장한다.<ref>{{서적 인용 |저자= Stephen T. Thornton|공저자= Jerry B. Marion|제목= Classical Dynamics of Particles and Systems|연도=2003|출판사= Brooks/Cole|판=Fifth Edition|쪽=p. 119|장=3.6 Sinusoidal Driving Forces}}</ref> 외력이 주기적으로 주어지는 강제조화진동의 경우, 운동의 동차해는 감쇠 진동의 해이기 때문에 점차 사라지는 '''과도적인 해'''({{lang|en|transient solution}})이기 때문에 오랜 시간이 지나면 특수해만이 남기 때문이다. 때문에 이 해를 '''정상상태 해'''({{lang|en|steady-state solution}})라 부르기도 한다.
 
==== 외력이 일반적인 힘일 때 ====
초기에 정지해 있는 진동의 경우, 다음과 같은 [[그린 함수]]를 사용해 문제를 해결할 수 있다.<ref>{{서적 인용 |저자= Stephen T. Thornton|공저자= Jerry B. Marion|제목= Classical Dynamics of Particles and Systems|연도=2003|출판사= Brooks/Cole|판=Fifth Edition|쪽=p. 136|장=3.9 The Response of Linear Oscillators to Impulsive Forcing functions (Optional), Response to an Impluse Fuction}}</ref><ref>문희태(2006), 《개정판 고전역학》, 서울 : 서울대학교출판부, 191쪽</ref>
:<math>
G(t,t') =
\begin{cases}
{ e^{-\lambda (t -t') } \over m \omega_1 } \sin \omega_1 (t - t') & t \ge t'
\\
0 & t < t'
\end{cases}
</math>
이때, 운동의 해는 다음과 같다.
:<math> x(t) = \int_{-\infty}^ t F(t') G(t,t') dt' </math>
 
== 동등한 계들 ==
여러 공학 분야에서 조화 진동자의 운동 방정식과 동등한 미분 방정식이 등장한다. 이러면, 그 [[계 (물리학)|계]]의 행동은 조화 진동자와 같은 행동을 보이게 된다. 아래는 기계, 전자 분야에서 등장하는 네 가지 예와 그에 해당하는 각각의 물리량을 비교하고 있다. 여기서 같은 줄에 있는 물리량들은 수학적으로 조화 진동자 모형 아래에서 동등한 물리량임을 의미한다.
<center>
{|class="wikitable" cellpadding="4" align="center"
!width="225"|용수철의 진동
!width="225"|비틀림 진자
!width="225"|직렬 [[RLC 회로]]
!width="225"|병렬 [[RLC 회로]]
|-
|위치 <math>x</math>||각 <math>\theta</math>||[[전하]] <math>q</math>||[[전압]] <math>V</math>
|-
|[[속도]] <math>\frac{dx}{dt}</math>
|[[각속도]] <math>\frac{d\theta}{dt}</math>
|[[전류]] <math>i = \frac{dq}{dt}</math>
|<math>\frac{dV}{dt}</math>
|-
|[[질량]] <math>m</math>||[[관성 모멘트]] <math>I</math>||[[인덕턴스]] <math>L</math> ||[[전기 용량]] <math>C</math>
|-
|[[훅 법칙|용수철 상수]] <math>k</math>
|[[비틀림 상수]](Torsion spring) <math>\mu</math>
|[[탄성율]](Elastance) <math>\frac 1 C</math>
|[[서셉턴스]] <math>\frac 1 L</math>
|-
|[[마찰계수]] <math>b</math>||[[회전 마찰계수]](Rotational friction ) <math>\Gamma</math>||[[전기 저항|저항]] <math>R</math>||[[전도율]] <math>\frac 1 R</math>
|-
|[[외력]] <math>F(t)</math>||[[돌림힘]] <math>\tau(t)</math>||<math>\frac {dV} {dt}</math>||<math>\frac {di}{dt}</math>
|-
|colspan="4" align="center"|감쇠가 없을때의 [[공명|공명 진동수]] <math>f_n</math>:
|-
|align="center"|<math>\frac{1}{2\pi}\sqrt{\frac{k}{M}}</math>
|align="center"|<math>\frac{1}{2\pi}\sqrt{\frac{\mu}{I}}</math>
|align="center"|<math>\frac{1}{2\pi}\sqrt{\frac{1}{LC}}</math>
|align="center"|<math>\frac{1}{2\pi}\sqrt{\frac{1}{LC}}</math>
|-
|colspan="4" align="center"|[[미분 방정식]]:
|-
|align="center"|<math>m\ddot x + b\dot x + kx = F</math>
|align="center"|<math>I\ddot \theta + \Gamma\dot \theta + \mu \theta = \tau</math>
|align="center"|<math>L\ddot q + R\dot q + \frac q C = V</math>
|align="center"|<math>C\ddot V + \frac {\dot V} R + \frac V L = q</math>
|}
</center>
 
== 참고 ==
{{각주}}
 
== 참고 문헌 ==
* 문희태(2006), 《개정판 고전역학》, 서울 : 서울대학교출판부.
* {{서적 인용 |저자= Stephen T Thornton|공저자= Jerry B. Marion|제목= Classical Dynamics of Particles and Systems|연도=2003|출판사= Brooks/Cole|판=Fifth Edition}}
* {{서적 인용 |저자=Herbert Goldstein|공저자=Charles Poole, John Safko|제목= Classical Mechanics|연도=2002|출판사=Addison Wesley|판=Third Edition}}
* {{서적 인용 |저자= Dennis. G. Zill|공저자= Michael R. Cullen|제목= Advanced Engineering Mathematics|연도=2006|출판사= Jones & Bartlett Pub|판=Third Edition}}
 
== 같이 보기 ==
{{위키공용|Category:Harmonic oscillation|조화 진동}}
{{위키공용|Category:Harmonic oscillator|조화 진동자}}
 
* [[감쇠비]]
* [[등속원운동]]
* [[비조화성]] (Anharmonicity)
* [[양자 조화 진동자]]
* [[임계속도]] (위험속도,Critical speed)
* [[진동]]
* [[진자]]
* [[페이저 (전자)]]
* [[Q 인자]]
* [[RC 회로]]
* [[RLC 회로]]
 
 
[[분류:고전역학]]